Difference between revisions of "1986 AIME Problems/Problem 4"
(→Solution) |
m (→Solution 2) |
||
(10 intermediate revisions by one other user not shown) | |||
Line 11: | Line 11: | ||
== Solution 2 == | == Solution 2 == | ||
+ | |||
+ | Subtracting the first equation from every one of the other equations yields | ||
+ | <cmath>\begin{align*} | ||
+ | x_2-x_1&=6\\ | ||
+ | x_3-x_1&=18\\ | ||
+ | x_4-x_1&=42\\ | ||
+ | x_5-x_1&=90 | ||
+ | \end{align*}</cmath> | ||
+ | Thus | ||
+ | <cmath>\begin{align*} | ||
+ | 2x_1+x_2+x_3+x_4+x_5&=6\\ | ||
+ | 2x_1+(x_1+6)+(x_1+18)+(x_1+42)+(x_1+90)&=6\\ | ||
+ | 6x_1+156&=6\\ | ||
+ | x_1&=-25 | ||
+ | \end{align*}</cmath> | ||
+ | Using the previous equations, | ||
+ | <cmath>3x_4+2x_5=3(x_1+42)+2(x_1+90)=\boxed{181}</cmath> | ||
+ | |||
+ | ~ Nafer | ||
== See also == | == See also == |
Latest revision as of 15:25, 17 November 2019
Contents
Problem
Determine if , , , , and satisfy the system of equations below.
Solution
Adding all five equations gives us so . Subtracting this from the fourth given equation gives and subtracting it from the fifth given equation gives , so our answer is .
Solution 2
Subtracting the first equation from every one of the other equations yields Thus Using the previous equations,
~ Nafer
See also
1986 AIME (Problems • Answer Key • Resources) | ||
Preceded by Problem 3 |
Followed by Problem 5 | |
1 • 2 • 3 • 4 • 5 • 6 • 7 • 8 • 9 • 10 • 11 • 12 • 13 • 14 • 15 | ||
All AIME Problems and Solutions |
- AIME Problems and Solutions
- American Invitational Mathematics Examination
- Mathematics competition resources
The problems on this page are copyrighted by the Mathematical Association of America's American Mathematics Competitions.